Sei sulla pagina 1di 22

PRINCIPLES OF MATHEMATICAL ANALYSIS.

WALTER RUDIN

these solutions were typed at warp speed, often with little or no preparation. And very little proofreading. Consequently, there are bound to be loads of mistakes. Consider it part of the challenge of the course to nd these errors. (And when you do nd some, please let me know so I can x them!) 7. Sequences and Series of Functions 1. Prove that every uniformly convergent sequence of bounded functions is uniformly bounded. {fn } is uniformly Cauchy, so |fn (x) fm (x)| < 1, for n, m bigger than some large N . Then |fn (x)| |fN (x)| |fn (x) fN (x)| < 1, so |fn (x)| < |fN (x)| + 1. Let M be a bound for fN . Then |fn (x)| < M + 1, so {fn } is uniformly bounded by M + 1. Dene n=N K := max{sup |f1 |, sup |f2 |, . . . , sup |fN 1 |, M + 1}. Then {fn } is uniformly bounded by K. n=1 2. Show that {fn }, {gn } converge uniformly on E implies {fn + gn } converges uniformly on E. If, in addition, {fn }, {gn } are sequences of bounded functions, prove that {fn gn } converges uniformly. sup{|(fn + gn )(x) (f + g)(x)|} = sup{|fn (x) f (x) + gn (x) g(x)|}
xE xE

Disclaimer:

sup{|fn (x) f (x)| + |gn (x) g(x)|}


xE xE

sup |fn (x) f (x)| + sup |gn (x) g(x)|


xE

0 + 0. When {fn }, {gn } are bounded sequences, sup{|(fn gn )(x) (f g)(x)|}


xE

March 23, 2006. Solutions by Erin P. J. Pearse.

Principles of Mathematical Analysis Walter Rudin = sup{|fn gn (x) f gn (x) + f gn (x) f g(x)|}
xE

sup{|fn (x) f (x)||gn (x)| + |f (x)||gn (x) g(x)|}


xE xE

sup |fn (x) f (x)||gn (x)| + sup |f (x)||gn (x) g(x)|


xE

sup |fn (x) f (x)| Mg + sup Mf |gn (x) g(x)|.


xE xE

We can introduce the uniform bounds Mf and Mg by problem 1 and the additional hypothesis. Then it is clear that the last line goes to 0 as n . 3. Construct sequences {fn }, {gn } which converge uniformly on a set E, but such that {fn gn } does not converge uniformly. 1 x 1 Work on I = (0, 1). Let fn (x) = x + n and gn (x) = 1x 1x so that n gn is the horizontal reection of fn , translated 1 to the right. (Graph them to see it.) fn (x) =
1 x

x n


1x n

1 x

= f (x)

0x1

x sup |fn f | = sup n = xI

1 n

0
1 n

1 gn (x) = 1x

1 1x = g(x)

0x1

sup |gn g| = sup 1x = n


xI

(n + (x 1)2 )(n + x2 ) n 1 = f g(x) 2 x(x 1) n x x2 x2 (x 1)2 + n + 2nx(x 1) = , for any n. sup |fn gn f g| = sup n2 x(x 1) 0x1 xI To see the sup is innite, check x = 0, 1. fn gn (x) = 4. Consider the sum f (x) =
n=1

1 . 1 + n2 x

For what x does the series converge absolutely? On what intervals does it converge uniformly? On what intervals does it fail to converge uniformly? Is f bounded? For xk = k12 , we get

f (xk ) =
n=1

1 1

. n 2 k

The k th term of the sum is undened, so f is undened at xk = k12 , k = 1, 2, . . . To be completed.

Solutions by Erin P. J. Pearse 5. Dene a sequence of functions by 0 fn (x) = sin2 x 0

x<

1 < n+1 1 <x n

1 n+1

, x .

1 n

Show that the series {fn } converges to a continuous function, but not uniformly. Use the series fn to show that absolute convergence, even for all x, does not imply uniform convergence. For x 0, fn (x) = 0 for every n, so lim fn (x) = 0. For x > 0, n > N :=
pw 1 x

1 n

< x = fn (x) = 0.

Thus fn (x) f (x) := 0 for x R. To see that the convergence is not uniform, consider fn (x) = 2 sin x cos x
x2 .

fn (x) = 0 when 1 sin = 0, in which case x = k for some k Z, or x 2 cos = 0, in which case x = 2k+1 for some k Z. x For each fn , only a few of these values occur where fn is not dened to be 0, so checking these values of x, fn
1 n

= sin(n) = 0 = sin((n + 1)) = 0 = sin


2n+1 2

1 fn n+1 2 fn 2n+1

= 1.

So Mn = supx {|fn (x) f (x)|} = 1 for each n, and clearly Mn 1 = 0. The series fn (x) converges absolutely for all x R: for any xed x, n=1 there is only one nonzero term in the sum. The series fn (x) does not converge uniformly: check partial sums. n=1
N N +1

sup
x n=1

fn (x)
n=1

fn (x) = sup |fN +1 (x)| = 1,


x

so the sequence of partial sums is not Cauchy (in the topology of uniform convergence), hence cannot converge. 6. Prove that the series

(1)n
n=1

x2 + n n2

converges uniformly in every bounded interval, but does not converge absolutely for any value of x. To see uniform convergence on a bounded interval,
a<x<b

sup {|fN (x) f (x)|} = sup

n=N +1

a<x<b

(1)n x n+n 2

Principles of Mathematical Analysis Walter Rudin =


n=N +1 +n (1)n c n2
2

(7.1)

where c := max{|a|, |b|}. We will use the alternating series test to show n c2 +n converges. From this, it will follow that (7.1) goes to 0 as n=1 (1) n2 N . 2 (i) For all x R, x n+n > 0. So the sum alternates. 2 x2 +n 1 (ii) For xed x, lim n2 = lim 2n = 0. (LHp) o (iii) To check monotonicity, prove x2 + n + 1 x2 + n (n + 1)2 n2 by cross-multiplying.
x 7. For n = 1, 2, 3, . . . , and x R, put fn (x) = 1+nx2 . Show that {fn } converges uniformly to a function f , and that the equation f (x) = limn fn (x) is correct if x = 0 but false if x = 0.

8. If I(x) = 0 (x 0), 1 (x > 0), |cn | converges, then

if {xn } is a sequence of distinct points in (a, b), and if prove that the series

f (x) =
n=1

cn I(x xn )

(a x b)

converges uniformly, and that f is continuous for every x = xn .

Solutions by Erin P. J. Pearse 9. {fn } are continuous and fn f on E. Show lim fn (xn ) = f (x) for every sequence {xn } E with xn x. Pick N1 such that n N1 = sup |fn (x) f (x)| < /2.
xE unif

Then surely |fn (xn ) f (xn )| < /2 holds for each n N1 . By Thm. 7.12, f is continuous, so pick N2 such that n N2 = |f (xn ) f (x)| < /2. Then we are done because |fn (xn ) f (x)| |fn (xn ) f (xn )| + |f (xn ) f (x)|. 11. {fn }, {gn } are dened on E with: unif (a) { N fn } uniformly bounded, (b) gn g on E, and (c) gn (x) n=1 gn1 (x) x E, n. Prove fn gn converges uniformly on E. Dene a := supxE |fn (x)| and bn := supxE |gn (x)|. Then

sup
xE N +1

fn (x)gn (x)
N +1

sup |fn (x)gn (x)|


xE N +1

an bn 0,

by Thm. 3.42. 13. {fn } is monotonically increasing on R, and 0 fn (x) 1 (a) Show f, {nk } such that f (x) = limk fnk (x), x R. By Thm. 7.23, we can nd a subsequence {fni } such that {fni (r)} converges for every rational r. Thus we may dene f (x) := suprx f (r), where the supremum is taken over r Q. It is clear that f is monotone, because x < y = {r x} {r y} and the supremum can only increase on a larger set. Thus, f has at most a countable set of discontinuities, by Thm. 4.30, pick x such that f is continuous at x. We want to show fni (x) f (x). Fix > 0. Since f is continuous at x, choose such that |x y| < = |f (x) f (y)| < /3. Now pick a rational number r [x 3 , x]. Then |fni (x) f (x)| |fni (x) fni (r)| + |fni (r) f (r)| + |f (r) f (x)|. (7.2) On the RHS of (7.2): the last term is less than /3 by the choice of ; and the middle term is less than /3 whenever i N1 for some large N1 , because the subsequence converges on the rationals. It remains to show the rst term gets small.

Principles of Mathematical Analysis Walter Rudin Pick some rational s [x, x + /3]. Then r x s and the continuity of f at x shows |r s| < = |f (r) f (s)| < /3. Also, since the fni are monotone, fni (r) fni (x) fni (s). With |fni (r) fni (s)| |fni (r) f (r)| + |f (r) f (s)| + |f (s) fni (s)|, some large N2 , i N2 gives |fni (r) fni (s)| < . By (7.3), this shows |fni (r) fni (x)| < . (b) If f is continuous, show fnk f uniformly on compact sets. Let K be compact. Fix > 0. Since f is uniformly continuous on K, pick such that |x y| < = |f (x) f (y)| < /3. Since K is compact, we can nd {x1 , . . . xJ } such that K J B (xj ), where j=1 B (xj ) := (xj , xj + ). 16. {fn } is equicontinuous on a compact set K, fn f on K. Prove {fn } converges uniformly on K. Dene f by f (x) := lim fn (x). Fix . From equicontinuity, nd such that |x y| < = |fn (x) fn (y)| < /3, n, x, y. Letting n , this gives |x y| < = |f (x) f (y)| < /3, x, y. Since K is compact, we can choose a nite set {x1 , . . . , xJ } such that K
J j=1 pw

(7.3)

B (xj )

where B (xj ) := (xj , xj + ). For each xj , we know fn (xj ) f (xj ), so pick N big enough that n N = |fn (xj ) f (xj )| < /3, j = 1, . . . , J. For any x K, x B (xj ) for some j. Thus for all x K, |fn (x) f (x)| |fn (x) fn (xj )| + |fn (xj ) f (xj )| + |f (xj ) f (x)|.

Solutions by Erin P. J. Pearse 18. Let {fn } be uniformly bounded and Fn (x) := a f (t) dt for x [a, b]. Prove {Fnk } which converges uniformly on [a, b] We need to show {Fn } is equicontinuous. Then by Thm. 6.20, each Fn is continuous; and by Thm. 7.25(b), were done. So x > 0, let x < y, and let M be the uniform bound on the {fn }.
y y x

|Fn (x) Fn (y)| =


x

fn (t) dt
x

|fn (t)| dt M (y x)

Then pick any < /M and |x y| < = |Fn (x) Fn (y)| so {Fn } is equicontinuous. n,

Principles of Mathematical Analysis Walter Rudin 20. f is continuous on [0, 1] and 0 f (x)xn dx = 0, n = 0, 1, 2, . . . . Prove that f (x) 0. Let g be any polynomial. Then g(x) = a0 + a1 x + a2 x2 + + aK xK . By linearity of the integral,
1 K 1 K 1

f (x)g(x) dx =
0 k=0

ak
0

f (x)x dx =
k=0

0 = 0.

By the Weierstrass theorem, let {fn } be a sequence of polynomials which n=1 converge uniformly to f on [0, 1]. Then
1 0

f 2 (x) dx = lim

f (x)fn (x) dx = lim 0 = 0.


0 n

Then by Chap. 6, Exercise 2, f 2 (x) 0. Thus f (x) 0. 21. Let K be the unit circle in C and dene
N

A :=

f (e ) =
n=0

cn ein ... cn C, R .

To see that A separates points and vanishes at no point, note that A contains the identity function f (ei ) = ei . To see that there are continuous functions on K that are not in the uniform closure of A, note that
2 0

f (ei )ei d = 0

f A,

(7.4)

and hence for g = lim gn (uniform limit) with gn A,


2 0

g(ei )ei d = lim

2 0

gn (ei )ei d = 0,

by Thm. 7.16. Thus, all functions in the closure of A satisfy (7.4). However, if we choose an h which is not in A, like h(ei ) = ei , then h is clearly continuous on K, and
2 0

h(e )e d =
0

1 d = 2.

Thus h is not in the uniform closure of A.

Solutions by Erin P. J. Pearse 22. Assume f R() on [a, b] and prove that there are polynomials Pn such that
b n

lim

|f Pn |2 d = 0.
n

We need to nd {Pn } such that f Pn 2 0. Fix > 0. By Chap. 6, Exercise 12, we can nd g C[a, b] such that f g 2 < /2. Note that
b a

|g P |2
a

sup |g P |2 = sup |g P |2 (b a).

Then by the Weierstrass theorem, we can nd a polynomial P such that gP


2

sup |g P |(b a) < /2.


2

By Chap. 6, Exercise 11, this gives f P

< .

2 23. Put P0 = 0 and dene Pn+1 (x) := Pn (x) + (x2 Pn (x)) /2 for n = 0, 1, 2, . . . . Prove that limn Pn (x) = |x| uniformly on [1, 1]. Note that if Pn is even, the denition will force Pn+1 to be even, also. Now 2 P1 = x , so assume 0 Pn1 1 for |x| 1. Then 2 2 x2 Pn1 x2 Pn1 Pn = Pn1 + = + Pn1 1 2 2 2 2

By elementary calculus, 0y1 = f (y) = y(1 y ) 2


x2 2 1 takes values in [0, 2 ].

(7.5)

Since |x| 1 also implies 0

[0, 1], this gives 0 Pn 1. Then

|x| + Pn (x) 1 2 |x| + Pn (x) 01 1. 2 To see Pn (x) |x|, consider that for x 0, the inequality x P1 (x) = x x2 /2 0

(7.6)

holds in virtue of the positivity of f in (7.5). Then by the symmetry of even functions, this is true for |x| 1. Now suppose Pn1 |x|, i.e., |x|Pn1 (x) 0. Then the given identity, and (7.6), give |x| Pn = (|x| Pn1 ) 1 1 (|x| + Pn1 ) 0. 2 We have established 0 Pn (x) Pn+1 (x) |x| for 1 x 1. Now, for n = 1, it is clear that |x| P1 (x) = |x|
|x|2 2 |x| 2 1

(7.7)

|x| 1

Principles of Mathematical Analysis Walter Rudin So suppose |x| Pn1 (x) |x| 1 1
|x| 2 |x| 2 n1

. Then, multiplying each side by

Pn1 (x) 2

and using the identity, we get


|x| 2

(|x| Pn1 (x)) 1

Pn1 (x) 2

|x| 1

|x| 2 |x| 2 |x| 2

n1 n1 n

1 1

|x| 2 |x| 2

Pn1 (x) 2

|x| Pn (x) |x| 1 |x| Pn (x) |x| 1 Now consider gn (x) = x 1 fn (x) = |x| 1
|x| 2 n x n 2

.
x n1 2

on [0, 1]. gn (x) = 1


2 ; n+1

(n+1)x 2

shows that gn has extrema at 2 and


2 fn n+1 = 2 n+1

only the latter is in [0, 1]. Then for

, fn has extrema
n n n+1 |x|2 x

<
n

2 . n+1

We have established |x| Pn (x) |x| 1 Now sup |x| Pn (x) <
2 n+1 n

<

2 . n+1 unif

(7.8)

0 and Pn |x|.

10

Solutions by Erin P. J. Pearse 8. Some Special Functions


2 7. If 0 < x < , prove that < sin x < 1. 2 x 2 To see the rst inequality, suppose x0 (0, ) with sin x0 0 . Since 2 x 2 2 limx0 sin x = 1, IVT gives an y with = sin y . Dene g(x) = sin x x so x y 2 g(y) = 0. Then g (x) = cos(x) and g (x) = sin x < 0 for x in the interval. By IVT again, there is a point z (0, y) with g (z) = 0, so g (x) < 0 for x (y, ). Then g(y) = 0 implies that g( ) < 0. < 2 2 To see the second inequality, put f (x) = x sin x. Then f (x) = 1 cos x shows f (0) = 0 and f (x) > 0 for 0 < x < . 2

8. For n = 0, 1, 2, . . . and x R, prove | sin nx| n| sin x|. This is clearly true (with equality) for n = 0, 1, so induct on n; suppose | sin nx| n| sin x|. We could use this assumption, to say | sin(n + 1)x| = | sin(nx + x)| | sin nx| + | sin x| = (n + 1)| sin x|, if we could prove the central inequality. From the denitions of C(x) and S(x) given in (46), sin(x + y) = sin x cos y + cos x sin y. Applying this, | sin(nx + x)| | sin nx cos x| + | cos nx sin x| | sin nx| + | sin x|, since | cos x| 1 for all x. 9. (a) Put sN = 1 + 1 + + 2 Note that
N 1 . N

Prove that = limN (sN log N ) exists.


1

= lim

k=1

1 log N = k

1 1 [x] x

dx.

The following sum telescopes:


N

k=1

1 (log(k + 1) log k) = sN log(N + 1), k

so rewrite the summand as 1 1 ak := log 1 + k k We will bound ak by 1 x2 . Note that for 2

x2 x + log(1 + x), f (x) = 2

11

Principles of Mathematical Analysis Walter Rudin


1 1 we have f (x) = x 1 + 1+x and f (x) = 1 = (1+x)2 . This gives f (0) = 0 and f (x) > 0 for all x > 0, so that f is always positive for x > 0 and

0 x log(1 + x)

x2 . 2
1 k2

1 In particular, this is true for x = k . Since N N

converges, this shows

lim

ak = lim
k=1

sN log(N + 1)

exists. Finally, log(N + 1) log N = log 1 + 1 N log 1 = 0


N

shows that limN (sN log N ) = 0. (b) Roughly how large must m be such that N = 10m satises sN > 100? From the above, and problem #13, we have 2 2 0 sN log N = log N sN log N + . 6 6 m Then m 43.43 will ensure log 10 > 100. 12. Suppose that f is periodic with f (x) = f (x + 2), and for (0, ), f (x) = 1, |x| , 0, < |x| < .

(a) Compute the Fourier coecients of f . 1 cn = 2


einx dx =

1 sin n ein ein = . 2in n

(b) Conclude that

n=1

sin n = , n 2 =
nZ nZ

(0 < < ). sin n n sin n . n

2
n=1

cn =

nZ

12

Solutions by Erin P. J. Pearse (c) Deduce from Parsevals theorem that

n=1

sin2 n = . 2 n 2 1 2

Parsevals theorem gives equality between 1 2 and |cn | =


nZ nZ 2

|f (x)|2 dx =

1 dx =

sin2 n = 2 2 n2 1 2

nZ

sin2 n . n2

Then

|cn |2 =
n=1

c0

Note that c0 =

also.

(d) Let 0 and prove that


0

sin x x

dx =

. 2

(e) Put = /2 in (c). What do you get? 2

n=0

sin2 ( n) 2 = 2 n 4

=
k=0

1 2 = . (2k + 1)2 8

13. Put f (x) = x for 0 x < 2, and apply Parsevals Theorem to conclude

n=1

1 2 = . n2 6

Apply it to the 2-periodic function f (x) = x on (, ) instead. Integration by parts gives cn = 1 2


xeinx dx =

(1)n , in 2 . 3

so with Parsevals theorem, 1 = n2 |cn |2 =


nZ

nZ

1 2

|f (x)|2 dx =

Since we have |cn | = |cn |, we nd the desired series by subtracting 1 c0 = 2


x dx = 1 [x2 ] = 0 2

13

Principles of Mathematical Analysis Walter Rudin from each side, and dividing the remainder by 2.

n=1

1 1 = 2 n 2

|cn |2 c0
nZ

1 2

2 0 3

2 . 6

9. Functions of Several Variables 5. Prove that to every A L(Rn , R) there corresponds a unique y Rn such that Ax = x y. For N = {x Rn ... Ax = 0}, N is a closed subspace by Problem 4. If N = Rn , then Ax = 0 x x, and were done. So suppose N = Rn . Then there is a nonzero vector x0 N = {u Rn ... u v = 0, v N }. We check Ax = yA x for yA dened by yA = Ax0 x0 . x0 2 Ax0 x0 x0 = yA x0 . x0 2

First, if x N , then Ax = 0 = yA x. Next, if x = x0 , then Ax = A(x0 ) = Ax0 =

Since the functions A(x) and yA x are linear functions of x and agree on N and x0 , they must agree on the space spanned by N and x0 . But N and x0 span Rn , since every element y Rn can be written y= y Ay Ay x0 + x0 . Ax0 Ax0

Thus Ax = yA x for all x Rn . If Ax = y x also, then y yA


2

= A(y yA ) A(y yA ) = 0. x = yA , and

So y = yA is unique. Equality of the norms comes from the inequalities: A = sup |Ax| = sup |yA x| sup yA
x 1 x 1 x 1

A = sup |Ax| A
x 1

yA yA

= yA

yA = yA . yA

6. If f (0, 0) = 0 and f (x1 , x2 ) = x2 1 x1 x2 , + x2 2 for (x1 , x2 ) = 0,

prove that D1 f (x1 , x2 ) and D2 f (x1 , x2 ) exist for every point (x1 , x2 ) R2 , although f is not continuous at (0, 0).

14

Solutions by Erin P. J. Pearse First, the derivatives are D1 f (x1 , x2 ) = x2 (x2 x2 ) 2 1 , (x2 + x2 )2 1 2 and D2 f (x1 , x2 ) = x1 (x2 x2 ) 1 2 , (x2 + x2 )2 1 2

and so clearly exist wherever (x, y) = (0, 0). To check the origin, consider that along the axes, 0 0 D1 f (x1 , 0) = 4 = 0, and D2 f (0, x2 ) = 4 = 0. x1 x2 However, for f to be continuous at (0, 0), we must have f (0, 0) =
(x,y)(0,0)

lim

f (x, y),

no matter how (x, y) (0, 0)! Dene (t) : R R2 by (t) = (x(t), y(t)) = (t, t), so that we approach the origin along the diagonal. Then lim f (x(t), y(t)) = lim f (t, t) = lim
t0 t0

t2 1 1 = lim = = 0. 2 t0 2t t0 2 2

7. Suppose that f is a R-valued dened in an open set E Rn , and that the partial derivatives D1 f, . . . , Dn f are bounded in E. Prove that f is continuous in E. Take m = 1 as in 9.21. Pick x E and consider the ball of radius r B(x, r) E. Choose h Rn such that h < r. Dene vk Rn by vk = (h1 , . . . , hk , 0, . . . , 0) for k = 1, . . . , n. Then as in (42),
n

f (x + h) f (x) =
j=1

[f (x + vj ) f (x + vj1 )] .

Since |vk | < r for 1 k n, and since B = B(x, r) is convex, the segments with endpoints x + vj1 and x + vj lie in B. Then vj = vj1 + hj ej , where hj ej = (0, . . . , 0, hj , 0, . . . , 0). Then the Mean Value Theorem applies to the partials and shows that the j th summand is hj (Dj f )(x + vj1 + tj hj ej ) for some tj (0, 1). By hypothesis, we have M such that |(Dj f )(x)| M
n

j = 1, . . . , n, x E.

Applying this to the absolute value of the above dierence, |f (x + h) f (x)|


j=1

|hj | M nM max{|hj |} nM h .

15

Principles of Mathematical Analysis Walter Rudin 8. Suppose that f is a dierentiable real function in an open set E Rn , and that f has a local maximum at a point x E. Prove that f (x) = 0. Let {ej } be the standard basis vectors of Rn , and dene j (t) = f (x0 + tej ), for t R. Then j : R R is dierentiable, so by Thm. 5.8, j (0) = (Dj f )(x0 ) = 0. But the partial derivatives (Dj f )(x0 ) are precisely the columns of the matrix (Df )(x0 ) = f (x). 9. If f is a dierentiable mapping of a connected open set E Rn into Rm , and in f (x) = 0 for every x E, prove f is constant in E. First, suppose m = 1. Pick some x0 E and dene C = {x E ... f (x) = f (x0 )}, D = {x E ... f (x) = f (x0 )} = E \ C. Clearly, C D = E and C D = . If we can show each of C, D is open, then D must be empty, or else wed have a disconnection of the connected set E. E is open, so for any x C, pick > 0 such that B(x, ) E. Then for any y B(x, ), the segment [x, y] B(x, ). Since f (x) = 0, we have f (x) = f (y) by Thm. 5.11(b). Also, we chose x C, so f (x) = f (y) = f (x0 ). This puts B(x, ) C and shows C is open. Now suppose we have some x E \ C. Since E is open, we can again choose > 0 such that B(x, ) E. By the exact same argument, we get y B(x, ) = f (x) = f (y) = f (x0 ), so that B(x, ) D and D is open. Finally, if m > 1, then apply this argument to each component of f . 10. Suppose f is a dierentiable mapping of R into R3 such that |f (t)| = 1 for every t. Prove f (t) f (t) = 0. Interpret this geometrically. Since f (t) = 1, we have (f1 (t))2 + (f2 (t))2 + (f3 (t))2 = 1. Dierentiating both sides, 2f1 (t)f1 (t) + 2f2 (t)f2 (t) + 2f3 (t)f3 (t) = 0 f (t) f (t) = 0. This means that for any curve on the unit sphere, the tangent at p S 1 is orthogonal to p, i.e., the surface of a sphere is orthogonal to the radius at any point.

16

Solutions by Erin P. J. Pearse 14. Dene f (0, 0) = 0 and f (x, y) = x3 /(x2 + y 2 ) for (x, y) = (0, 0).
2 1 0

-1

-2

-1

-2 2 1 0 -1 -2 2

(a) Prove that D1 f and D2 f are bounded functions in R2 so that f is continuous. We have the partial derivatives x4 + 3x2 y 2 , and x4 + 2x2 y 2 + y 4 2x3 y D2 f = 4 . x + 2x2 y 2 + y 4 D1 f = Boundedness at : 3(x4 + 3x2 y 2 ) = 3, as |x| , and x4 + 2x2 y 2 |2x3 | |y| |D2 f | = 4 0. 2y + y3| |x /y + 2x |D1 f | Boundedness away from : only need to check zeroes of the denominators, and there is only (0, 0). f (x, 0) f (0, 0) lim 1 = 1, and x0 x0 x f (0, y) f (0, 0) D2 f (0, 0) = lim lim 0 = 0. y0 y0 y D1 f (0, 0) = lim Thus f is continuous by Prob. 7. (b) Let u R2 , |u| = 1. Show (Du f )(0, 0) exists and |(Du f )(0, 0)| 1. The directional derivative is f [(0, 0) + tu] f (0, 0) 1 t 3 x3 Du f (0, 0) = lim = lim 2 2 t0 t0 t t (x + y 2 ) t x3 = 2 , x + y2

17

Principles of Mathematical Analysis Walter Rudin for u = (x, y). Since |u| = x2 + y 2 = 1, this implies |x| 1. Hence we have Du f (0, 0) = x3 and |Du f (0, 0)| = |x3 | 1.

(c) Let be a dierentiable mapping of R into R2 with (0) = (0, 0) and | (0)| > 0. Put g(t) = f ((t)) and prove that g is dierentiable for every t R. Put (t) = (x(t), y(t)) so that x(t)3 g(t) = , x(t)2 + y(t)2 for t = 0.

Since is dierentiable for t = 0, we have x(t), y(t) dierentiable for t = 0, and hence g(t) is dierentiable for t = 0, by the chain rule, Thm. 9.15.1 It remains to check that g is dierentiable at the origin. g (0) = lim g(h) g(0) h0 h 1 x(h)3 = lim h0 h x(h)2 + y(h)2 x(h)3 h2 = lim h0 h3 x(h)2 + y(h)2 = x(h) lim h0 h
3 3

h2 h0 x(h)2 + y(h)2 lim

= (x (0))

1 . | (0)|2

The last two lines are easiest to see by working backwards. Also show C = g C . Note that g (t) = f ((t)) (t). Since the additional hypothesis is that (t) is continuous (which is equivalent to saying x(t), y(t) are continuous, by Thm. 4.10), we just need that f ((t)) is continuous. Since the chain rule gives g (t) = x(t)2 (x(t)2 x (t) + 3y(t)2 x (t) 2x(t)y(t)y (t)) , (x(t)2 + y(t)2 )2

it is clear that g (t) is continuous whenever x, y are not simultaneously 0. For t = 0 (or other t0 s.t. x(t0 ) = y(t0 ) = 0), replace x(t) by x (0)t + o(t) (using Taylors Thm.) and similarly replace y(t) by y (0)t + o(t). Thus g (t)
1By (a), (x (0)t+o(t))2 ((x (0)t+o(t))2 x (t)+3(y (0)t+o(t))2 x (t)2(x (0)t+o(t))(y (0)t+o(t))y (t)) ((x (0)t+o(t))2 +(y (0)t+o(t))2 )2

the partials of f exist and are continuous in an open nbd of (0, 0). Hence, f is continuously dierentiable at (0, 0) by Thm. 9.21.

18

Solutions by Erin P. J. Pearse so that g (t) g (0). (d) In spite of this, f is not dierentiable at (0, 0). Let u = (x, y) be a unit vector. By Rudins (40), Du = D1 f (0, 0)x + D2 f (0, 0)y = 1 x + 0 y = x. But from (b) we have Du = x3 = x for any x other than 1, 0, 1. (Bonus problem) This problem isnt in Rudin. Dene f : R2 R by f (x, y) = 0,
x2 y , x4 +y 2 t0

(x, y) = (0, 0), otherwise.

Let a be any straight line through the origin with slope a and a (0) = (0, 0). This means that (up to parametrization) a (t) = (t, at) for some a R, or (t) = (0, t). Show that lim f (a (t)) = f (0, 0) = 0,
t0

but that f is not continuous at the origin.

-1 -0.5 0 0.5 -1 1 0 1

0.5 0.25 0 -0.25 -0.5

Continuity of the restriction follows by a basic computation: at3 at lim f (a (t)) = lim 4 = lim 2 = 0, and 2 t0 t0 t + (at) t0 t + a2 0 = 0, lim f ( (t)) = lim t0 t0 0 + t2 To see the other part, approach the origin along a parabolic curve: let (t) = (x(t), y(t)) = (t, t2 ). Then lim f ((t)) = lim
t0

t4 1 1 t2 t2 = lim 4 = lim = = 0. 4 + (t2 )2 t0 2t t0 2 t0 t 2

19

Principles of Mathematical Analysis Walter Rudin 15. Dene f (x, y) = x2 + y 2 2x2 y


4x6 y 2 . (x4 +y 2 )2

2 20 10 0 -10 -2

-1 -2

-1 0 1 2

(a) Prove that 4x4 y 2 (x4 + y 2 )2 , so that f C 0 . (x4 + y 2 )2 4x4 y 2 = x8 + 2x4 y 2 + y 4 4x4 y 2 = (x4 y 2 )2 0 Now f (x, y) = x2 + y 2 2x2 y x2 (x, y), where 0 (x, y) 1. This gives f continuous at (0, 0). (b) For 0 < , t R, let g (t) = f (t cos , t sin ). Show that g (0) = 0, g (0) = 0, g (0) = 2. Each g thus has a strict local minimum at t = 0, i.e., on each line through (0, 0), f has a strict local minimum at (0, 0). Since sin , cos are bounded, g (0) = f (0, 0) = 0 by (a). By some nightmarishly tortuous (but elementary) calculation, get the other two results. (c) Show that (0, 0) is not a local minimum of f , since f (x, x2 ) = x4 . Substituting in y = x2 : f (x, x2 ) = x2 + x4 2x4 = x2 x4 x2 = x4 . 16. Dene f (t) = t + 2t2 sin 1 , f (0) = 0. Then f (0) = 1, f bounded in (1, 1), t but f is not injective in any nbd of 0. 4x10 (2x4 )2

20

Solutions by Erin P. J. Pearse


0.2 0.1

-0.2

-0.1 -0.1 -0.2

0.1

0.2

First note that f (t) = 1 2 cos are bounded by 1, we have |f (t)| 1 + 4 + 2 = 7, At t = 0, we have that

1 t

+ 4t sin

1 t

for t = 0. Since sin t, cos t

for t (1, 1) \ {0}.

t + 2t2 sin 1 f (t) f (0) t = = 1 + 2t sin 1 t t t shows f (0) = 1 by the Sandwich theorem applied to t + 2t2 and t 2t2 . So f is bounded in all of (1, 1). 1 Dene sn = 2n for n = 1, 2, . . . , so that f (sn ) = 1 2 cos(2n) + 2 Then dene tn =
2 (4n+3)

sin 2n = 1 2 = 1. n for n = 1, 2, . . . , so that


8 (4n+3)

f (tn ) = 1 2 cos (4n+3) + 2

sin (4n+3) = 1 + 2

8 (4n+3)

> 0.

Since f changes sign between each successive sn and tn , and since sn , tn 0, f fails to be injective in every neighbourhood of 0. 17. Let f = (f1 , f2 ) : R2 R2 be given by f1 (x, y) = ex cos y, (a) What is the range of f ? Note that if we identify (0, 1) = f2 (x, y) = ex sin y.

1 = i, then

f (z) = f (x + iy) = ez = ex+iy = ex cos y + iex sin y. But you only really need to see that (ex cos y, ex sin y) is polar coordinates for a point with radius ex and argument y, to see that the range is any point of R2 with radius r > 0, i.e., R2 \ (0, 0).

21

Principles of Mathematical Analysis Walter Rudin (b) Show that the Jacobian of f is nonzero. Thus, every point of R2 has a neighbourhood in which f is injective. However, f is not injective. The Jacobian is the determinant of partials: ex cos y ex sin y ex sin y ex cos x = e2x (cos2 x + sin2 x) = e2x = 0.

But f is not injective, since f (x, y) = f (x, y + 2n), for n Z. (c) Put a = (0, ) and b = f (a). Let g be the continuous inverse of f , 3 dened in a neighbourhood of b, such that g(b) = a. Find an explicit formula for g, compute f (a) and g (b), and verify Rudins (52). For u = ex cos y, v = ex sin y, one veries that x = 1 log(u2 + v 2 ) = log r, 2 y = tan1
v u

for r = ex , and
v for = u . 1 2

= arg ,
1 2 3 2

For the derivatives, f (a) = and g (u, v) = Finally, f (g(u, v)) = f = = = log
1 u2 +v 2

3 2

v u

g (b) =

1 2 23

3 2 1 2

u2 + v 2 , tan1 cos tan1 sin


u u2 +v 2 v u2 +v 2

v u

u2 + v 2 u2 + v 2 u v v u
1

v u v tan1 u

sin tan1 cos tan1


v u

v u

u2v+v2
u u2 +v2

= [g (u, v)]

(d) What are the images under f of lines parallel to the coordinate axes? Lines parallel to the x-axis are mapped to straight lines through the origin, parameterized exponentially. Lines parallel to the y-axis are circles about the origin of radius ex , parameterized with constant speed. Diagonal lines (t) = (at + b, btc ) will get mapped to (et cos t, et sin t), counterclockwise logarithmic spirals emanating from the origin.

22

Potrebbero piacerti anche